(10,-5)(10-9) what is the linear equation

Answers

Answer 1

In this case, we'll have to carry out several steps to find the solution.

Step 01:

Data

point 1 (10 , - 5) x1 = 10 y1 = -5

point 2 (10, - 9) x2 = 10 y2 = -9

linear equation​ = ?

Step 02:

Slope formula

[tex]m\text{ = }\frac{y2\text{ -y1}}{x2-x1}[/tex][tex]m\text{ = }\frac{-9-(-5)}{10-10}=\frac{-9+5}{0}=\frac{-4}{0}=\infty[/tex]

The answer is:

The equation of the line is a vertical line in x = 10, since the slope is infinite.


Related Questions

five to the third power

Answers

We need to find the value of 5 to the third power, to do this let's remember that the third power of a number means multiplying this number three times by itself, that is:

[tex]5^3=5\times5\times5=125[/tex]

The answer is 125

Jacob distributed a survey to his fellow students asking them how many hours they'd spent playing sports in the past day. He also asked them to rate their mood on a scale from 0 to 10, with 10 being the happiest. A line was fit to the data to model the relationship.Which of these linear equations best describes the given model?A) ŷ = 5x + 1.5B) ŷ = 1.5x + 5Or C) ŷ = -1.5x + 5Based on this equation, estimate the mood rating for a student that spent 2.5 hours playing sports.Round your answer to the nearest hundredth.__________.

Answers

We have to relate a linear function (the regression model) with its equation.

We can see in the graph that the y-intercept, the value of y(0), is b=5.

Then, we can estimate the slope with the known points (0,5) and (2,8):

[tex]m=\frac{y_2-y_1}{x_2-x_1}=\frac{8-5}{2-0}=\frac{3}{2}=1.5[/tex]

Then, with slope m=1.5 and b=5, the regression model equation should be:

[tex]y=1.5x+5[/tex]

We can estimate the mood for students that spent 2.5 hours playing sports by replacing x with 2.5 in the model and calculate y:

[tex]y(2.5)=1.5\cdot2.5+5=3.75+5=8.75[/tex]

NOTE: we could also have look on the graph instead of doing the calculation.

Answer: B) y=1.5x+5

The estimation of the mood for a student that spent 2.5 hours playing sports is 8.75.

Every week Ben collects a few pounds of paper to recycle. The graph below shows the total number of pounds of paper(y) that Ben collected in a certain amount of time (x), in weeks:

Answers

To obtain the amount of paper that would most likely be collected in 10 weeks, the following steps are necessary:

Step 1: Select two points that lie on the straight line and use the two points to derive the equation of the straight line, as follows:

Such two points could be: (x1, y1) = (0, 30) and (x2, y2) = (120, 3)

Using the following formula, we can derive the equation of the straight line:

[tex]\frac{y-y_1}{x-x_1}=\frac{y_2-y_1}{x_2-x_1}[/tex]

Thus:

[tex]\begin{gathered} \frac{y-y_1}{x-x_1}=\frac{y_2-y_1}{x_2-x_1} \\ \Rightarrow\frac{y-30_{}}{x-0_{}}=\frac{120_{}-30_{}}{3_{}-0_{}} \\ \Rightarrow\frac{y-30_{}}{x_{}}=\frac{90_{}}{3_{}_{}} \\ \Rightarrow\frac{y-30_{}}{x_{}}=30 \\ \Rightarrow y-30=30\times x \\ \Rightarrow y=30x+30 \end{gathered}[/tex]

The above equation can be re-written as:

[tex]\begin{gathered} y=30x+30 \\ \Rightarrow\text{Amount of paper collected = 30 }\times number\text{ of w}eeks\text{ + 30 } \end{gathered}[/tex]

Step 2: Use the derived equation to obtain the value of the amount of paper collected in 10 weeks, as follows:

In 10 weeks, we will have :

[tex]\begin{gathered} \text{Amount of paper collected = 30 }\times number\text{ of w}eeks\text{ + 30 } \\ \Rightarrow\text{Amount of paper collected = 30 }\times10\text{ + 30 } \\ \Rightarrow\text{Amount of paper collected = 300 + 30 }=330 \\ \Rightarrow\text{Amount of paper collected = 3}30 \end{gathered}[/tex]

Therefore, the amount of paper that would most likely be collected in 10 weeks is 330

at 37 ft string of lights will be attached to the top of a 35 ft pole for Holiday display how far from the base of a pool should the end of the string of lights be anchored

Answers

Answer:

12 feet

Explanation:

The diagram representing the problem is attached below:

The distance of the pole to the base of the string is the value x.

Using Pythagoras Theorem:

[tex]\begin{gathered} 37^2=35^2+x^2 \\ x^2=37^2-35^2 \\ x^2=144 \\ x^2=12^2 \\ x=12ft \end{gathered}[/tex]

The end of the string should be anchored 12 ft from the base of the pole.

DreviousRandom numbers are useful forA creatingOB. beingOC. modelingOD. sellingReset Selectionreal-world situations that involve chance.

Answers

Given random numbers, it is important to remember that Random Numbers are defined as those numbers that each have the same probability of being selected.

In Statistics, random numbers are useful to model different real-world situations.

An example of random numbers is the numbers of a lottery.

Another example of random numbers is the numbers obtained by rolling a numbered dice.

Hence, the answer is: Option C.

two trains leave town 906 miles aprt at the same time and travel each other. one train travels 21 mi/h faster than other. if they meet in 6 hours, what is the rate of each train?

Answers

Let

x be the speed of the slower train

x + 21 be the speed of the faster train

They meet in 6 hours which means that their speed is

[tex]\frac{906}{6}=151[/tex]

The sum of their therefore is

[tex]\begin{gathered} x+(x+21)=151 \\ 2x+21=151 \\ 2x=151-21 \\ 2x=130 \\ \frac{2x}{2}=\frac{130}{2} \\ x=65\frac{\operatorname{mi}}{\operatorname{h}}\text{ (speed of slower train)} \\ \\ x+21=65+21=86\frac{\operatorname{mi}}{\operatorname{h}}\text{ (speed of faster train)} \end{gathered}[/tex]

Pythagorean Theorem Task Card #1 A pool table is 9 feet long and 5 feet wide. How far is it from one corner pocket to the diagonally opposite corner pocket? Round to the nearest tenth.

Answers

A right triangle is made where the two legs are 9 ft (a) and 5 ft (b), and we have to find the hypotenuse (c). Using Pythagorean Theorem:

c² = a² + b²

c² = 9² + 5²

c² = 81 + 25

c = √106

c ≈ 10.3 ft

01 Question 11 What is the area of the shaded region? 12cm 6 cm 10cm 8cm 128cm? 96cm2 X 144cm? a 112cm?

Answers

We can calculate the area of the shaded region as the difference between the area of the rectangle (sides 12 cm and 10 cm) and the area of the triangle in the corner.

The triangle has base of b=12-8=4 cm and height h=10-6=4 cm.

Then, we can calculate the area of the shaded area as:

[tex]\begin{gathered} A=A_r-A_t \\ A=b_r\cdot h_r-\frac{b_t\cdot h_t}{2} \\ A=12\cdot10-\frac{4\cdot4}{2} \\ A=120-\frac{16}{2} \\ A=120-8 \\ A=112\operatorname{cm}^2 \end{gathered}[/tex]

Answer: 112 cm^2

10. Linek is graphed below. Write an equation for line m that is perpendicular to line (there are multiple correct answers).

Answers

step 1

Find the equation of line k

the slope of line k is

m=-3/4 ------> previous answer

step 2

If two lines are perpendicular, then their slopes are opposite reciprocal

so

the slope of the line m must be equal to

m=4/3

I will assume a point (3,4)

Find the equation in slope intercept form

y=mx+b

we have

m=4/3

point (3,4)

substitute

4=(4/3)*(3)+b

solve for b

4=4+b

b=0

therefore

y=(4/3)x -----> equation in slope intercept form ( this line is perpendicular to line k)

Find the equation in point slope form

y-y1=m(x-x1)

we have

m=4/3

point (3,4)

substitute

y-4=(4/3)*(x-3) -----> equation in point slope form ( this line is perpendicular to line k)

Find the equation in standard form

Ax+By=C

where

A is a positive integer

B and C are integers

we have

y=(4/3)x

Multiply by 3 both sides

3y=4x

4x-3y=0 ------> equation in standard formy=(4/3)xy-4=(4/3)*(x-3)4x-3y=0

The Rainforest Pyramid at Moody Gardens is a rectangular pyramid that has glass sides to allow the sunlight to enter the building. The base of the building is approximately 320 feet wide and 200 feet long. The height of each side is approximately 140 feet. How many square feet of glass were needed to cover the lateral sides of the pyramid?

Answers

We have the following diagram for a rectangular pyramid:

Where:

• h is the height of the pyramid,

,

• s is the slant height, the height of the sides.

The area of the sides (without the base) of the pyramid is given by:

[tex]A=\frac{1}{2}\cdot p\cdot s\text{.}[/tex]

Where p is the perimeter of the base.

We have a rectangular pyramid with:

• base with sides a = 320 ft and b = 200 ft,

,

• slant height (heigh of the sides ) s = 140 ft.

The perimeter of the base is:

[tex]p=2\cdot(a+b)=2\cdot(320ft+200ft)=1040ft\text{.}[/tex]

Replacing the value of p and s in the formula of the area we get:

[tex]A=\frac{1}{2}\cdot(1040ft)\cdot140ft=72800ft^2.[/tex]

Answer

It was needed 72800 ft² of glass.

The table gives a set of outcomes and their probabilities. Let A be the event "the outcom less than or equal to 2". Let B be the event "the outcome is a divisor of 3". Find P(AB). Outcome Probability 1 0.2 2 0.6 3 0.2

Answers

Given the table:

Outcome Probability

1 0.2

2 0.6

3 0.2

Let A be the event "the outcome less than or equal to 2.

Let B be the event "the outcome is a divisor of 3.

Let's find P(A ∩ B).

To find P(A ∩ B) let's first find P(A) and P(B).

Numbers less than or equal to 2 = 2 and 1

Outcomes less than or equal to 2 = P(2) or P(1)

Probability the outcome is less than or equal to 2 = P(A) = 0.2 + 0.6 = 0.8

Divisor of 3 = 1 and 3

Outcome is a divisor of 3 = P(1) and P(3)

Probability the outcome is a divisor of 3 = P(B) = 0.2 x 0.2 = 0.04

To find P(A ∩ B), we have:

P(A ∩ B) = P(A) x P(B) = 0.04 x 0.8 = 0.032

ANSWER:

0.032

The table lists recommended amounts of food to order for 25 party guests. Amanda and Syndey are hosting a graduation party for 40 guests. They know there will also be guests stopping by who may have come from other parties. For ordering purposes, they will count each of these "drop-in" guests as half a guest. How much of each food item should Amanda and Syndey order for a graduation party with 45 drop-in guests?

Answers

Amanda and Sydney are hosting a graduation party for 40 guests.

Also, they have 45 drop-in guests (each of these will count as half a guest).

Then, the total number of guests is:

[tex]40+\frac{45}{2}=\frac{40\times2+1\times45}{1\times2}=\frac{80+45}{2}=\frac{125}{2}[/tex]

The table shows the recommended amounts of food for 25 party guests:

Fried Chicken: 24 pieces

Deli meats: 4 1/3 pounds

Lasagna: 10 3/4 pounds.

Let's find the proportion for 125/2 guests:

a. Fried chicken:

[tex]\begin{gathered} \frac{24\text{ pieces}}{25\text{ guests}}=\frac{x\text{ pieces}}{125/2\text{ guests}} \\ \text{Set the product of the diagonals equal to each other:} \\ 24\times\frac{125}{2}=x\times25 \\ \frac{24\times125}{2}=x\times25 \\ 1500=x\times25 \\ \text{Divide both sides by 25} \\ \frac{1500}{25}=\frac{x\times25}{25} \\ \text{Simplify} \\ 60=x \end{gathered}[/tex]

Then, they'll need to order 60 units of fried chicken for the party.

b. Deli meats:

Start by converting the mixed number 4 1/3 into a fraction:

[tex]4\frac{1}{3}=\frac{4\times3+1}{3}=\frac{12+1}{3}=\frac{13}{3}[/tex]

Now, apply proportions:

[tex]\begin{gathered} \frac{13/3\text{ pounds}}{25\text{ guests}}=\frac{x\text{ pounds}}{125/2\text{ guests}} \\ \text{Set the product of the diagonals equal to each other:} \\ x\times25=\frac{13}{3}\times\frac{125}{2} \\ x\times25=\frac{13\times125}{3\times2} \\ x\times25=\frac{1625}{6} \\ \text{Divide both sides by 25} \\ \frac{x\times25}{25}=\frac{1625}{6\times25}=\frac{1625}{150}=\frac{65}{6} \\ \text{Simplify} \\ x=\frac{65}{6} \end{gathered}[/tex]

You also can convert this fraction into a mixed number:

[tex]\begin{gathered} \text{When you divide 65/6 you obtain a quotient of 10 and remainder 5.} \\ \text{Then the whole part is 10 and the fraction is 5/6} \\ \frac{65}{6}=10\frac{5}{6} \end{gathered}[/tex]

Then, they'll need to order 10 5/6 pounds of deli meats for the party.

c. Lasagna

Convert the mixed number to fraction:

[tex]10\frac{3}{4}=\frac{10\times4+3}{4}=\frac{40+3}{4}=\frac{43}{4}[/tex]

Apply proportions:

[tex]\begin{gathered} \frac{43/4\text{ pounds}}{25\text{ guests}}=\frac{x\text{ pounds}}{125/2\text{ guests}} \\ \text{Set the product of the diagonals equal to each other:} \\ x\times25=\frac{43}{4}\times\frac{125}{2} \\ x\times25=\frac{43\times125}{4\times2} \\ x\times25=\frac{5375}{8} \\ \text{Divide both sides by 25} \\ \frac{x\times25}{25}=\frac{5375}{8\times25}=\frac{5375}{200}=\frac{215}{8} \\ \text{Simplify} \\ x=\frac{215}{8} \end{gathered}[/tex]

You also can convert this fraction into a mixed number:

[tex]\begin{gathered} \text{When you divide 215/8 you obtain a quotient of 26 and remainder 7.} \\ \text{Then the whole part is 26 and the fraction is 7/}8 \\ \frac{215}{8}=26\frac{7}{8} \end{gathered}[/tex]

Then, they'll need to order 26 7/8 pounds of lasagna for the party.

Find the exact value of cos -1050.OA.-3OB.-12/2OC. 1OD. 1/3Reset Selection

Answers

Solution:

Given;

[tex]\cos(-1050)[/tex]

Rewrite the expression using;

[tex]\cos(-x)=\cos x[/tex]

Thus;

[tex]\begin{gathered} \cos(-1050)=\cos(1050) \\ \\ \cos(1050)=\cos(330) \end{gathered}[/tex]

Then;

[tex]\cos(330)=\frac{\sqrt{3}}{2}[/tex]

CORRECT OPTION: D

3x-22x + 1A8If x=8, then the length of AB is

Answers

Ok the length of AB is given by the equation:

[tex]3x-2[/tex]

If x=8 then we simply have to replace this value in the equation:

[tex]\bar{AB}=3\cdot8-2=22[/tex]

And that's the length of segment AB.

Find the principal P that corresponds to the future value F= $1,300 under r = 4% interest compounded daily for t = 3 years. Round your final answer to two decimal places.

Answers

The formula to find the principal is:

[tex]FV=P(1+i)^n[/tex]

Where

FV is future value

P is principal

i is the rate of interest

n is the time frame

Given,

FV = 1300

i = r = 4%, or 4/100 = 0.04

n = t = 3

Plugging into formula and solving gives us:

[tex]\begin{gathered} FV=P(1+i)^n \\ 1300=P(1+0.04)^3 \\ 1300=P(1.04)^3 \\ P=\frac{1300}{1.04^3} \\ P=1155.695 \end{gathered}[/tex]

Rounding to 2 decimal places:

$1155.70

m/cour16121/quizzes/2919544/take22A totem pole casts a shadow 45 feet long at the same time that a 6-foot-tallmancasts a shadow that is 3 feet long. Find the height of the totem pole.Height of the totemfeetHint: set a proportion like this oneshadow totemshadow manactual height totem actual height manucation

Answers

The totem pole with its shadow form a right triangle, and the same happens with the man and its shadow. Since it happens at the same time of the day, the "inclination" of the sunlight is the same for both, and those two triangles are similar, which means that their corresponding ratiso betweens sides are the same.

Using the hint given on the problem, we can estabilish the following equation for the height of the totem(let's call it h).

[tex]\frac{45}{h}=\frac{3}{6}[/tex]

Now, we just need to solve for h.

[tex]\begin{gathered} \frac{45}{h}=\frac{3}{6} \\ \frac{45}{h}=\frac{1}{2} \\ \frac{h}{45}=2 \\ h=2\cdot45 \\ h=90 \end{gathered}[/tex]

The height of the totem pole is 90 ft.

Simplify (sqrt)98m^12 using factor tree or splitting up using perfect squares. Quick answer showing work = amazing review :)

Answers

The expression is:

[tex]\sqrt[]{98m^{12}}[/tex]

We need to use a factor tree to solve the problem. We will draw it as shown below:

According to the factor tree we can represent the 98 as:

[tex]\sqrt[]{2\cdot7^2m^{12}}[/tex]

We can now remove the terms that have a power of 2 and a power of 12. For that we need to divide the exponents by 2.

[tex]7^{}\sqrt[]{2}m^6[/tex]

The simplified expression is 7*sqrt(2)*m^6.

which of the following represents a line that is parallel to the line with equation y = – 3x + 4 ?A. 6x +2y = 15B. 3x - y = 7 C. 2x - 3y = 6D. x + 3y = 1

Answers

In order to have parallel lines, the slopes of the lines need to be the same.

In order to check the slope for each option, let's put the equations in the slope-intercept form:

[tex]y=mx+b[/tex]

Where m is the slope and b is the y-intercept.

So we have that:

A.

[tex]\begin{gathered} 6x+2y=15 \\ 2y=-6x+15 \\ y=-3x+7.5 \end{gathered}[/tex]

B.

[tex]\begin{gathered} 3x-y=7 \\ y=3x-7 \end{gathered}[/tex]

C.

[tex]\begin{gathered} 2x-3y=6 \\ 3y=2x-6 \\ y=\frac{2}{3}x-2 \end{gathered}[/tex]

D.

[tex]\begin{gathered} x+3y=1 \\ 3y=-x+1 \\ y=-\frac{1}{3}x+\frac{1}{3} \end{gathered}[/tex]

So the only option with a line with a slope of -3 is Option A.

Fnd the volume of each cylinder below.9.18 in15 in

Answers

9) We can calculate the volume as the product of the base area and the height.

The base is a circle with radius r=18 in. Then, its area is:

[tex]A_b=\pi r^2=\pi\cdot18^2=324\pi[/tex]

Then, we can calculate the volume V as:

[tex]V=A_b\cdot h=324\pi\cdot15=4860\pi[/tex]

10) In this case the circular base is on the side, but we can still use the same principle to calculate the volume.

The area of the base with diameter D = 11 in is:

[tex]A_b=\frac{\pi D^2}{4}=\frac{\pi\cdot11^2}{4}=\frac{\pi\cdot121}{4}=\frac{121}{4}\pi[/tex]

Then, we can calculate the volume V as:

[tex]V=A_b\cdot h=\frac{121}{4}\pi\cdot21=\frac{2541}{4}\pi=635.25\pi[/tex]

Answer:

9) V = 4860π

10) V = 635.25π

ХI went to the bank 4 times last week and withdrew a total of $160. What was theaverage amount withdrew each time? Write and solve an expression to find theanswer

Answers

[tex]\begin{gathered} \text{ since we went to the bank }4\text{ times, we will use the expression} \\ \\ \text{average}=\frac{total\text{ money}}{times\text{ we went to the bank}} \\ \\ \text{average}=\frac{160}{4} \\ \\ \text{average}=40 \\ \\ \text{the avegare amount we withdrew each time is \$40} \end{gathered}[/tex]

Watch help videoFind the value of y in the diagram below.Y +9Y + 9Y +9y +9Y +9116Answer: Submit Answer

Answers

The equation from the box obtained is

[tex]y+9+y+9+y+9+y+9+y+9=116[/tex][tex]5y+45=116[/tex][tex]5y=71[/tex][tex]y=14.2[/tex]

Select the correct choice and fill in the answer box

Answers

To begin with, let us look at a few definitions that will help

A relation is a function if each x-value is paired with exactly one y-value. A vertical line test on a graph can be used to determine whether a relation is a function.

If we use a graph to check, we will have

We can see that there is no overlapping of coordinates. The table satisfies the vertical line test.

Hence, it is a function

The domain and range of function is the set of all possible inputs and outputs of a function respectively. The domain and range of a function y = f(x) is given as domain= {x ,x∈R }, range= {f(x), x∈Domain}.

The domain of the function, D is given by

[tex]D=\mleft\lbrace-1,0,1,2,3\mright\rbrace[/tex]

The range, R is given by

[tex]R=\mleft\lbrace-6,-1,2,5,8\mright\rbrace[/tex]

A ball is shot out of a cannon at ground level. it's height H in feet after t seconds is given by the function H(t) = 96t - 16t^2. Find H(1), H(5), H(2), and H(4). Why are some of the outputs equal? H(1) = ______ feetH(2)= ______ feetH(4)= ______ feet H(5)= ______ feet

Answers

Follow the function we have that

[tex]\begin{gathered} H(1)=96(1)-16(1)^2 \\ =96-16=80 \end{gathered}[/tex]

So H(1) = 80 feet. Now

[tex]\begin{gathered} H(2)=96(2)-16(2)^2 \\ =192-64=128 \end{gathered}[/tex]

So H(2) = 128 feet. Now

[tex]\begin{gathered} H(4)=96(4)-16(4)^2 \\ =384-256=128 \end{gathered}[/tex]

So H(4) = 128 feet. Now

[tex]\begin{gathered} H(5)=96(5)-16(5)^2 \\ =480-400=80 \end{gathered}[/tex]

So H(5) = 80 feet.

Parallelogram ABCD was translated to parallelogram AB'C'D'.How many units and in which direction were the x-coordinates of parallelogram ABCD moved?

Answers

Answer:

Number of units: 7

Direction: left of the x coordinate

Explanation:

Given:

Parallelogram ABCD was translated to parallelogram AB'C'D'

To find:

the number of units and in which direction were the x-coordinates of parallelogram ABCD moved

To determine the number of units, we will use the diagram below:

From A to A', B to B', C to C' and D to D', the number of units for the movement in the x coordinate is 7. The vertical position didn't change. This means to movement in the coordinate.

The parallelogram ABCD was moved to the left of the x axis to get parallelogram A'B'C'D'

Number of units: 7

Direction: left of the x coordinate

7 units to the left

given the following trig equation, find the Exact value of the remaining 5 trig functionstan (theta) = 5/6 and cos theta < 0Start by drawing the triangle in standard position and use the Pythagorean theorem to find the remaining side. A. label the exact value of all 3 sides of the triangle drawn in the correct quadrantB. DETERMINE the EXACT value of the remaining 5 trig functions! (sin) (cos) (tan) (sec) (csc) (cot)

Answers

tan (theta) = 5/6 and cos theta < 0

tan (theta) = 5/6 ==> theta = tan^-1(5/6) = 39.80557109

theta = 39.80557109

cos(theta) = cos(39.80557109) = 0.7682212796

It says that cos(theta) < 0, so the 39.80557109 degrees is in rality an angle of 90 + 39.80557109 = 120.80557109

sin (theta) = sin (120.80557109) = 0.858910105

cos(theta) = cos(120.80557109) = −0.5121263824

tan(theta) = tan(120.80557109) = −1.677144811

sec(theta) = sec(120.80557109) = −1.952643008

csc(theta) = csc(120.80557109) = 1.164266195

cot(theta) = cot(120.80557109) = −0.5962514348

Sound is measured in decibels, using the formula d = 10 log() where P is the intensity of the sound and P, is the weakest sound the human ear can hear. A horn has a decibel warning of20. How many times more intense is this horn compared to the weakest sound heard to the human ear?

Answers

We have the following:

The formula is the following

[tex]d=10\log (\frac{P}{P_o})[/tex]

From what the statement tells us, the value of d is equal to 20, now we replace and solving for P / Po

[tex]\begin{gathered} 10\log (\frac{P}{P_o})=20 \\ \frac{10}{10}\log (\frac{P}{P_o})=\frac{20}{10} \\ \log (\frac{P}{P_o})=2\rightarrow\log (x)=2\rightarrow x=10^2\rightarrow x=100 \\ \frac{P}{P_o}=100 \\ P=100\cdot P_o \end{gathered}[/tex]

Therefore, the sound of the horn is about 100 more intense than the weakest sound heard to the human ear.

What is the value of u? H 88 U +64 I K к 88 50 J U =

Answers

HK = KJ

So:

u+64 = 5u

Solve for u

Combine like terms:

64 = 5u-u

64= 4u

Divide both sides by 4

64/4 =4u/4

16 = u

the product of 8 and a number increased by 17

Answers

You have the following stament:

The product of 8 and a number increased by 17​

In order to write the previous statment in an algebraic way, you take into account that "the product of 8 and a number" means the multiplication of 8 and a variable, which is "increased" by 17. That is, number 8 is multiplying the sum of a variable and 17.

Thus, you have:

The product of 8 and a number increased by 17​:

8(x + 17)

at the rate shown in the table how many books were sold in 3 weeks

Answers

210 books were sold in 3 weeks

Explanation

Step 1

you can easily solve this by using a rule of three

number of weeks=1

sold books=70

then, the rate is

[tex]\text{rate}=\frac{70\text{ books}}{1\text{ we}ek}=70\text{ book}s\text{ per w}eek[/tex]

Step 2

Let

x represents the number of books sold in 3 weeks,then the rate is

[tex]\text{rate}=\frac{x}{3\text{ w}eeks}[/tex]

as the rates are equal

[tex]\begin{gathered} 70=\frac{x}{3} \\ Multiply\text{ both sides by 3} \\ 70\cdot3=\frac{x}{3}\cdot3 \\ x=210 \end{gathered}[/tex]

I hope this helps you

Simply this expression 4(1-3x)+7x-8

Answers

Answer:

-5x-4

Step-by-step explanation:

4(1-3x)+7x-8

Distribute the 4

4(1)+4(-3x)+7x-8

4-12x+7x-8

Combine like terms

-12x+7x+4-8

-5x-4

hopes this helps please mark brainliest

Other Questions
The population P of Phoenix, Arizona (in thousands) from 1970 through 2000 can be modeled by the equation P = 590e ^ (0.027t) where r represents the year, with t = 0 corresponding to 1970. According to this model, when will the population reach 1.5 million. Simplify each expression. (5x2y)(3x4)a. 15x6yb. 15x2yc. 15x6yd. 15x8y owners of limited liability companies enjoy flow-through treatment of income and losses. group of answer choices true false Find the equation of a line perpendicular to y = -3x + 5 that passes through the point (6,7) Solve the equation 7x = 91 for x. 98 84 13 13 Identify all pairs of congruent corresponding parts.Then write another congruence statement for the polygons.AABC ~ADEF The Quadratic f(x)=x^2-2x-15Using the functions of your graphing calculator calculate the coordinates of the following points (as shown in the calculator videos in this lesson). If the parabola doesn't intersect the x-axis then write "none." If necessary, round to the nearest hundredths place (2 decimal places).a. The vertex using the min/max calculate function.b. X-intercept(s) using the zero calculate function.c. Y-intercept using the value calculate function (w/ a value of x=0).d. Now, copy down the t-table generated by your calculator for integer input values from-3x3. Please Help! Functions and Relations The graph shows the absolute value parent function. which statement best describes the function? Katie rents a car when spending her vacation in Argentina while she returns the car she has driven 900 miles and used about 36 gallons of gas if you guess cost an average of $4.139 Per gallon estimate how much she spent on fuel From the waiting area, they walked another 0.1 miles to board the plane. The plane left the gate 45 min after they arrived at the waiting area. Part C: what was the length from the waiting area to the airplanes takeoff? The slope of the line below is 2 Write the point-slope equation of the line using the coordinates of the labeled point. If I have an unknown quantity of gas at a pressure of 120 kPa, a volume of 19 liters, and a temperature of 35C, how many moles of gas do I have? (Convert temperature to K, and do not include the units in the answer) Samuel has 10 coins that add up to $3.80. Some are 50 cent and the rest are 20 cent coins. How many 50 cent coins does he have? will mark brainliest question in the picture How many red squares will there be if there are 60 squares? 19. Which of the following is equal to V-24 ?O-2iV64i 166i-1/2O21 V6 After Batman brought the Joker to justice, the crime rate in Arkham City decreased by 12\%12%12, percent. Previously, the crime rate was ccc incidents per 100010001000 people. HELP!! 100 POINTS AND BRAINLIEST!!!Choose three possible solutions to the problem. (immigration to the us). Briefly compare and contrast the advantages and disadvantages associated with these solutions. In the table below note three advantages and disadvantages for each solution. BE sure to use research to support your claims. As shown in Fig. B9, a block of mass m2=2.1 kg sits on an inclined plane with angle 35 degrees .The coefficients of static and kinetic friction between the block and the plane are unknown.The block is attached to a rope, which is hung over a frictionless and massless pulley and attached to a hanging mass m1.(a) The block my is on the verge of sliding, when the hanging mass is increased to m1=1.6kg.01. find the coefficient of static friction.As shown in Fig. Bob, an identical block (with mass m2) is placed on top of the block on the inclined plane. The hanging mass m1 remains unchanged. The two blocks on the inclined plane are sliding down the incline with the same acceleration a=0.31 m/s^2Find02.the coefficient of kinetic friction.03.what is the minimum value of the coefficient of static friction between the two identical blocks (m2) so that they move together? Find the answers to fill in blank 1. And blank 2.